Parametric Hacks (I have the solution but something has gone wrong)

  • Thread starter Thread starter Quantumpencil
  • Start date Start date
  • Tags Tags
    Parametric
Click For Summary
The discussion revolves around verifying Stokes' Theorem for the vector field F(x,y,z)=(z,x,y) over a specified region. Two parameterizations of the boundary were attempted, with the second yielding the correct integral value of Pi, while the first resulted in 1-Pi/4. The user initially believed both parameterizations were equivalent but later realized that the first only represented half of the boundary, specifically the portion where y>0. This discrepancy led to the conclusion that the two parameterizations are not equivalent, prompting a request for assistance in identifying the error in the first parameterization. The discussion highlights the importance of accurately representing the entire boundary in vector calculus problems.
Quantumpencil
Messages
96
Reaction score
0

Homework Statement

Check Stokes Theorem for the Vector Field F(x,y,z)=(z,x,y), on the region defined by z=x^2-y^2 and x^2+y^2<=1.


Attempt at Solution:

Should be pretty easy problem; I tried two parameterizations, one worked and the other didn't... but I feel like they are equivalent. They were as follows: The boundary of the surface, defined when x^2+y^2 = 1, can be given by the paremetrization:

r(t)=(t, Sqrt[1-t^2], 2t^2-1)
r(t)=(Cos [Theta], Sin[Theta], Cos[2Theta]

Then the line integral along this path Should just be... the integral dt, from 0 to 1, or the integral dTheta from 0 to 2Pi of the dot product of F(r(t)) and r'(t), where r'(t) is given by

r'(t)=(1, -t/Sqrt[1-t^2], 4t)
r'(t)=(-Sin[Theta], Cos[Theta], -2Sin[2Theta]

Now, using the second parameterization, I got the value of this integral to be Pi. This is the right Answer. Using the first, I got the answer 1-Pi/4... but aren't they exactly equivalent?

I also evaluated the Surface integral of the Curls ((Curl F = 1,1,1)), and there got Pi using both co-ordinate systems (The integrals running from -Sqrt[1-x^2], to Sqrt[1-x^2] dy and -1 t0 1 dx, and from -1 to 1 dr and 0 to 2Pi dTheta, respectively)

Could someone help me figure out what is wrong with my first parameterization?
 
Physics news on Phys.org
I didn't work this out in detail, but one thing I notice is that y= \sqrt{1- x^2} only gives half of boundary: the half with y> 0. The two parameterizations are NOT equivalent.
 
Question: A clock's minute hand has length 4 and its hour hand has length 3. What is the distance between the tips at the moment when it is increasing most rapidly?(Putnam Exam Question) Answer: Making assumption that both the hands moves at constant angular velocities, the answer is ## \sqrt{7} .## But don't you think this assumption is somewhat doubtful and wrong?

Similar threads

  • · Replies 3 ·
Replies
3
Views
2K
  • · Replies 8 ·
Replies
8
Views
2K
  • · Replies 6 ·
Replies
6
Views
1K
  • · Replies 19 ·
Replies
19
Views
2K
  • · Replies 1 ·
Replies
1
Views
2K
  • · Replies 6 ·
Replies
6
Views
2K
Replies
3
Views
2K
  • · Replies 11 ·
Replies
11
Views
3K
  • · Replies 3 ·
Replies
3
Views
2K
  • · Replies 2 ·
Replies
2
Views
2K